Difference between revisions of "1976 IMO Problems/Problem 4"

m (Solution 3)
m
 
(4 intermediate revisions by 2 users not shown)
Line 2: Line 2:
 
Determine the greatest number, who is the product of some positive integers, and the sum of these numbers is <math>1976.</math>
 
Determine the greatest number, who is the product of some positive integers, and the sum of these numbers is <math>1976.</math>
  
== Solution ==
+
 
 
=== Solution 1 ===
 
=== Solution 1 ===
 
Since <math>3*3=2*2*2+1</math>, 3's are more efficient than 2's. We try to prove that 3's are more efficient than anything:
 
Since <math>3*3=2*2*2+1</math>, 3's are more efficient than 2's. We try to prove that 3's are more efficient than anything:
Line 14: Line 14:
 
<math>\dfrac{1}{\sqrt[3]{3}-1}<x</math>
 
<math>\dfrac{1}{\sqrt[3]{3}-1}<x</math>
  
Thus we need to prove that <math>\dfrac{1}{\sqrt[3]{3}-1}<4</math>. Simplifying, we get <math>5<4\sqrt[3]{3}\Rightarrow 125<64*3=192</math>, which is true. Working backwards, we see that all <math>x</math> greater than 3 are less efficient than 3, so we try to use the most 3's as possible:
+
Thus we need to prove that <math>\dfrac{1}{\sqrt[3]{3}-1}<4</math>. Simplifying, we get <math>5<4\sqrt[3]{3}\Rightarrow 125<64*3=192</math>, which is true. Working backwards, we see that all <math>x</math> greater than 3 are less efficient than 3, so we never use anything greater than 3. Therefore we use as many 3s in groups of 2 as possible, and since the remainder when 1976 is divided by 6 is 2, we can use 658 3s and one 2.
  
 
<math>\dfrac{1976}{3}=658.6666</math>, so the greatest product is <math>\boxed{3^{658}\cdot 2}</math>.
 
<math>\dfrac{1976}{3}=658.6666</math>, so the greatest product is <math>\boxed{3^{658}\cdot 2}</math>.
  
===Solution 2===
+
===Solution 2===  
 +
(Non-rigorous)
 
We demonstrate heuristically that 3's are the most efficient.  
 
We demonstrate heuristically that 3's are the most efficient.  
 
As we know that the chosen numbers must be equal, by the AM-GM inequality, we wish to maximize <cmath> f(x) = \left(\frac{1976}{x}\right)^{x} </cmath>
 
As we know that the chosen numbers must be equal, by the AM-GM inequality, we wish to maximize <cmath> f(x) = \left(\frac{1976}{x}\right)^{x} </cmath>
Simple logarithmic differentiation shows that <math>\frac{S}{x} = e</math> maximizes the given function. As <math>e</math> is approximately 2.71828, we use 2's and 3's only. 3's are optimal, but we must use one 2.  
+
Simple logarithmic differentiation shows that <math>\frac{S}{x} = e</math> maximizes the given function. As <math>e</math> is approximately 2.71828, we use 2's and 3's only. 3's are optimal, but we must use one 2.
  
 
=== Solution 3 ===
 
=== Solution 3 ===

Latest revision as of 04:29, 2 January 2023

Problem

Determine the greatest number, who is the product of some positive integers, and the sum of these numbers is $1976.$


Solution 1

Since $3*3=2*2*2+1$, 3's are more efficient than 2's. We try to prove that 3's are more efficient than anything:

Let there be a positive integer $x$. If $3$ is more efficient than $x$, then $x^3<3^x$. We try to prove that all integers greater than 3 are less efficient than 3:

When $x$ increases by 1, then the RHS is multiplied by 3. The other side is multiplied by $\dfrac{(x+1)^3}{x^3}$, and we must prove that this is less than 3 for all $x$ greater than 3.

$\dfrac{(x+1)^3}{x^3}<3\Rightarrow \dfrac{x+1}{x}<\sqrt[3]{3}\Rightarrow 1<(\sqrt[3]{3}-1)x$

$\dfrac{1}{\sqrt[3]{3}-1}<x$

Thus we need to prove that $\dfrac{1}{\sqrt[3]{3}-1}<4$. Simplifying, we get $5<4\sqrt[3]{3}\Rightarrow 125<64*3=192$, which is true. Working backwards, we see that all $x$ greater than 3 are less efficient than 3, so we never use anything greater than 3. Therefore we use as many 3s in groups of 2 as possible, and since the remainder when 1976 is divided by 6 is 2, we can use 658 3s and one 2.

$\dfrac{1976}{3}=658.6666$, so the greatest product is $\boxed{3^{658}\cdot 2}$.

Solution 2

(Non-rigorous) We demonstrate heuristically that 3's are the most efficient. As we know that the chosen numbers must be equal, by the AM-GM inequality, we wish to maximize \[f(x) = \left(\frac{1976}{x}\right)^{x}\] Simple logarithmic differentiation shows that $\frac{S}{x} = e$ maximizes the given function. As $e$ is approximately 2.71828, we use 2's and 3's only. 3's are optimal, but we must use one 2.

Solution 3

Note: This solution uses the same strategy as Solution 1 (that having the largest possible number of three's is good), but approaches the proof in a different manner.

Let $f(S) \triangleq \prod_{x \in S} x$, and $g(S) = \sum_{x\in S} x$, where $S$ is a multiset. We fix $g(S) = 1976$, and aim to maximize $f(S)$. Since $3(x-3) > x$ for $x \geq 5$, we notice that $S$ must only contain the integers $1,2,3$ and $4$. We can replace any occurrences of $4$ in $S$ by replacing it with a couple of $2$'s, without changing $f(S)$ or $g(S)$, so we may assume that $S$ only contains the integers $1,2$ and $3$. We may further assume that $S$ contains at most one $1$, since any two $1$'s can be replaced by a $2$ without changing $g(S)$, but with an increase in $f(S)$. If $S$ contains exactly one $1$, then it must also contain at least one $2$ (since $1976 \equiv 2\;(\mathrm{mod }\;3)$). We can then replace this pair of a $1$ and a $2$ with a $3$, thus keeping $g(S)$ constant, and increasing $f(S)$. Now we may assume that $S$ contains only $2$'s and $3$'s.

Now, as observed in the last solution, any triplet of $2$'s can be replaced by a couple of $3$'s, with $g(S)$ constant, and an increase in $f(S)$. Thus, after repeating this operation, we will be left with at most two $2$'s. Since $g(S) = 1976$, and $1976 \equiv 2\;(\mathrm{mod }\;3)$, we therefore get that $S$ must have exactly one $2$ (since we already showed it consists only of $2$'s and $3$'s). Thus, we get $\boxed{f(S) = 2\cdot 3^{658}}$.

See also

1976 IMO (Problems) • Resources
Preceded by
Problem 3
1 2 3 4 5 6 Followed by
Problem 5
All IMO Problems and Solutions